Manipal Engineering Manipal Engineering Solved Paper-2012

  • question_answer
    If\[I(m,\,\,n)=\int_{0}^{1}{{{t}^{m}}}{{(1+t)}^{n}}dt\], then the expression for\[I(m,\,\,n)\]in terms of\[I(m+1,\,\,n-1)\]is

    A) \[\frac{{{2}^{n}}}{m+1}-\frac{n}{m+1}I(m+1,\,\,n-1)\]

    B) \[\frac{n}{m+1}I(m+1,\,\,n-1)\]

    C) \[\frac{{{2}^{n}}}{m+1}+\frac{n}{m+1}I(m+1,\,\,n-1)\]

    D) \[\frac{m}{m+1}I(m+1,\,\,n-1)\]

    Correct Answer: B

    Solution :

    Here,\[I(m,\,\,n)=\int_{0}^{1}{{{t}^{m}}}{{(1+t)}^{n}}dt\] We apply integration by parts, taking\[{{(1+t)}^{\pi }}\]as first and\[{{t}^{m}}\]as second function]                 \[I(m,\,\,n)=\left[ {{(1+t)}^{n}}\cdot \frac{{{t}^{m+1}}}{m+1} \right]_{0}^{1}\]                 \[-\int_{0}^{1}{n{{(1+t)}^{n-1}}\cdot \frac{{{t}^{m+1}}}{m+1}dt}\]                 \[=\frac{{{2}^{n}}}{m+1}-\frac{n}{m+1}\int_{0}^{1}{{{(1+t)}^{n-1}}\cdot {{t}^{m+1}}dt}\] \[\therefore \]  \[I(m,\,\,n)=\frac{{{2}^{n}}}{m+1}-\frac{n}{m+1}\cdot I(m+1,\,\,n-1)\]


You need to login to perform this action.
You will be redirected in 3 sec spinner